Solve the initial value problem?

Click For Summary
To solve the initial value problem (dy/dx) = (1) / (2√x) + 10 with y(1) = -1, the approach involves finding two functions, f(x) and g(x), where f'(x) = (1) / (2√x) and g'(x) = 10. This simplifies the problem into two easier integration tasks. The integration can be performed by applying the rules of differentiation in reverse, leading to f(x) = ∫(1/(2√x))dx and g(x) = ∫10dx. After determining f(x) and g(x), the final step is to combine these functions and apply the initial condition to find the specific solution for y(x).
femmed0ll
Messages
6
Reaction score
0
1) (dy/dx) = (1) / ( 2√x ) + 10 , y(1) = -1
2) (d^(2)r / dt^2) = ( 4 / t^3 ) ; (dr/dt) | t=1 = -5, r(1)=5

... i have no idea where to start.
what are the steps needed to figure out the answer...can someone please solve this for me! t
thank you so much.
 
Physics news on Phys.org
(1) So the problem here is that you need to find a function y(x) such that when you differentiate it, you get (1) / ( 2√x ) + 10, right? Recall that the operation of differentiating is linear. Meaning that for any two functions f,g, we have (f + g)'=f ' + g'. This observation may help you to simplify the problem. Indeed, if you can find two functions f(x), g(x) such that f '(x)=(1) / ( 2√x ) and g'(x)=10, then the solution to the problem will be y(x) = f(x) + g(x). Do you agree?

Ok, so we've split the difficult problem of finding a function y(x) such that y'(x) = (1) / ( 2√x ) + 10 into the 2 easier problems of finding functions f(x), g(x) such that f '(x)=(1) / ( 2√x ) and g'(x)=10.

This is just a matter of "differentiating backwards". You are used to finding the derivative of a given function. Here, you are given a function and must find it is the derivative of what function.

If you have practiced differentiating enough, then this should not be too hard. Start with finding g(x) as it is somewhat easier than finding f(x). Help yourself with a table listing the rules of differentiation if necessary.

Equivalently, if you know what an integral is and if you're familiar with the rules for solving them, then

f(x)=\int \frac{1}{2\sqrt{x}}dx

and

g(x)=\int 10dx
 
Last edited:
Question: A clock's minute hand has length 4 and its hour hand has length 3. What is the distance between the tips at the moment when it is increasing most rapidly?(Putnam Exam Question) Answer: Making assumption that both the hands moves at constant angular velocities, the answer is ## \sqrt{7} .## But don't you think this assumption is somewhat doubtful and wrong?

Similar threads

  • · Replies 8 ·
Replies
8
Views
1K
  • · Replies 1 ·
Replies
1
Views
1K
  • · Replies 11 ·
Replies
11
Views
3K
  • · Replies 2 ·
Replies
2
Views
1K
  • · Replies 15 ·
Replies
15
Views
2K
  • · Replies 10 ·
Replies
10
Views
1K
Replies
5
Views
2K
  • · Replies 1 ·
Replies
1
Views
1K
  • · Replies 2 ·
Replies
2
Views
1K
Replies
5
Views
2K